Good inequality...

This topic has expert replies
User avatar
Legendary Member
Posts: 934
Joined: Tue Nov 09, 2010 5:16 am
Location: AAMCHI MUMBAI LOCAL
Thanked: 63 times
Followed by:14 members

Good inequality...

by [email protected] » Mon May 07, 2012 12:30 am
"If x is negative, is x < -3 ?

(1) x^2 > 9
(2) x^3 < -9 "


I donot have the OA but when I solved the question, I got the answer as E.
IT IS TIME TO BEAT THE GMAT

LEARNING, APPLICATION AND TIMING IS THE FACT OF GMAT AND LIFE AS WELL... KEEP PLAYING!!!

Whenever you feel that my post really helped you to learn something new, please press on the 'THANK' button.

User avatar
Master | Next Rank: 500 Posts
Posts: 385
Joined: Mon Apr 16, 2012 8:40 am
Location: Pune, India
Thanked: 186 times
Followed by:29 members

by aneesh.kg » Mon May 07, 2012 12:53 am
Statement (1):
x^2 - 9 > 0
Using critical points method for x < 0
x < -3
(You will get the same answer even by plugging-in numbers)
SUFFICIENT

Statement(2):
x^3 + 9 < 0
Plug-in negative values. The inequality does not hold true for -1 and -2. It holds true first when x^3 = - 9.
x = - (9)^1/3
Since (-2)^3 = - 8 and (-3)^3 = - 27, and
-27 < - 9 < - 8,
x is between -2 and -3 and slightly less than - 2.
x = - 2.1 (approx)
Below this value, the statement will always hold true.

x < - 2.1

i.e., x can be less than - 3 or even greater than it (for the values between - 2.1 and - 3).

INSUFFICIENT.

[spoiler](A)[/spoiler] is the answer.

Traps in this problem:
(1)The problem does not mention that x is an integer, so some people will fall into the trap of assuming x as an integer and go with (D)
(2) x is negative. If you don't notice this you will go with (C).

What is the Critical Points method?
Read here:
https://www.beatthegmat.com/critical-poi ... tml#465861
Aneesh Bangia
GMAT Math Coach
[email protected]

GMATPad:
Facebook Page: https://www.facebook.com/GMATPad

GMAT/MBA Expert

User avatar
GMAT Instructor
Posts: 3835
Joined: Fri Apr 02, 2010 10:00 pm
Location: Milpitas, CA
Thanked: 1854 times
Followed by:523 members
GMAT Score:770

by Anurag@Gurome » Tue May 08, 2012 8:39 pm
[email protected] wrote:"If x is negative, is x < -3 ?

(1) x^2 > 9
(2) x^3 < -9 "


I donot have the OA but when I solved the question, I got the answer as E.
(1) x² > 9 implies x < -3 or x > 3
It is given that x is negative or x < 0, so x < -3; SUFFICIENT.

(2) x^3 < -9
If x = -3, then x^3 = -27. Here x is equal to -3 and not less than -3.
If x = -4, then x^3 = -64. Here x < -3.
No definite answer; NOT sufficient.

The correct answer is A.
Anurag Mairal, Ph.D., MBA
GMAT Expert, Admissions and Career Guidance
Gurome, Inc.
1-800-566-4043 (USA)

Join Our Facebook Groups
GMAT with Gurome
https://www.facebook.com/groups/272466352793633/
Admissions with Gurome
https://www.facebook.com/groups/461459690536574/
Career Advising with Gurome
https://www.facebook.com/groups/360435787349781/